PT73.S2.Q15 - marketing consultant: last year I predicted

strasvery-deletedstrasvery-deleted Free Trial Member
edited September 2016 in Logical Reasoning 161 karma
Hello,
I have a question about these two choices
1. the argument takes for granted that the higher sales of established products are due to effective advertising
2. the argument confuses a condition necessary for increasing product sales with a condition that will ensure increased sales

So I thought in the stimulusu says
lower sale(This season's sales...)→ill conceived(Thus...)
Then 1. says effective ad→higher sale
So this looks like contrapositive of what the stimulus says
(ill conveived=effective ad)

In 2. I thought it says higer sale→effective ad is actually the correct one, and because the stimulus says lower sale→ill conceived, it confuses the sufficient and necessary.

Why I cannot think this way?

Thanks
https://7sage.com/lsat_explanations/lsat-73-section-2-question-15/

Comments

  • nessa.k13.0nessa.k13.0 Inactive ⭐
    edited October 2016 4141 karma
    Hi @strasvery !
    D) "It takes for granted that the higher sales of established products are due to effective advertising"
    E) "it confuses a condition necessary for increasing product sales with a condition that will ensure increased sales"

    The reasons (premises) given in the stimulus for the conclusion that "the advertising campaign was ill conceived" are 1. "LRG ignored my [the Market Consultant's] predictions and took the advice of a competing consultant", (we don't know for sure that last year's ad campaign is from the same competing consultant but lets keep going), 2."sales are down" and 3. "LRG's new products are selling especially poorly" (due to "LRG's latest advertising campaign" which was predicted to be both "unpopular with customers" and "ineffective in promoting new products." We don't know that "unpopular" and "ineffective" are the same as "sales going down" and "selling especially poorly").

    D) What do we know about "established products"? Absolutely nothing----we cannot support that the argument "takes for granted that the higher sales of established products are due to effective advertising." We also know nothing about effective advertising. We have no such information though on sales of "established products" nor their "effective advertising". We could assume we do, but that would be erroneous. The stimulus only gives us information about "LRG's new products [that] are selling especially poorly".

    Example of why D doesn't work: It is possible that LRG released a ton of "new products" that are higher in number than the "established products" and those "new products" did not sell well, but the smaller amount of the "established product" sales increased while their total amount of sales resulted in low sales overall (because the amount of new products exceeded that of the established products or vice versa). This is just one example of why choosing D and saying that the flaw in the argument "takes for granted that the higher sales of established products are due to effective advertising" is wrong (there's a part to whole issue with the argument, but not specifically this kind). We'd have to assume that the Marketing Consultant is assuming that "due to effective advertising" of "established products" and their "higher sales," the "new products" that used "LRG's latest advertising campaign" which was predicted to be "unpopular with customers and ineffective in promoting new products" resulted in this season's sales figures showing "that sales are down" and that "LRG's new products are selling especially poorly. Thus, the advertising campaign was ill conceived." We cannot say anything about established products and we'd be connecting the support to a *prediction* which is not specifically given in support of the conclusion, but is context for the argument. I eliminated this answer because the stimulus does not support any claim about "established products" and the "effective advertising" of such products.

    E. The argument does not "confuse a necessary condition for increasing product sales with a condition that will ensure increased sales". I eliminated this answer as well. If we assume that the Marketing Consultant is reasoning that: if they (LRG) uses the latest ad campaign, then it (use of the latest ad campaign) will result in low sales--- we would be making bad assumptions from the stimulus. It is not explicitly stated that there is a cause/effect relationship between "low sales" and "LRG's latest advertising campaign" like it is explicitly stated in the prediction--which is context that "LRG's latest advertising campaign" will be "unpopular with customers and ineffective with promoting new products" ---remember this is not "sales" and we can't conflate the two different things. Also we don't even know if the "latest advertising campaign" from the prediction is the same or different as the one from the competing consultant in the premise. On top of bringing in these assumptions (remember the support for the conclusion that "the advertising campaign was ill conceived" is specifically because 1."LRG ignored my [Marketing Consultant] predictions and took the advice of a competing consultant," 2."sales are down," and 3. "new products are selling especially poorly" due to an ad campaign that was predicted to be "unpopular with customers" and "ineffective in promoting new products." Furthermore we have nothing to support the notion that the Marketing Consultant is good at their job and thus made a correct prediction-- WHEW back to the main sentence) we would also be making an error in misidentifying the necessary condition with answer choice E, because it stipulates that the argument is flawed in asserting that the Marketing Consultant "confuses a condition necessary for increasing product sales with a condition that will ensure increased sales".

    I hope this makes sense to you, but this is my detailed breakdown of why I eliminated both D and E. This question was MUCH easier to solve by honing in on specifically how the Marketing Consultant supports the conclusion that "the advertising campaign is ill conceived". I don't think mapping this question out using conditional logic would be the most effective approach to answering this question mainly because the argument is not stated in conditional language---doing so on this question will use up too much time.

    I hope^ that helps!
  • strasvery-deletedstrasvery-deleted Free Trial Member
    161 karma
    Thank you so much for the explanation! honestly after reading your explanation, I'm starting to think my review is not enough at all...
    Thanks again :)
  • nessa.k13.0nessa.k13.0 Inactive ⭐
    4141 karma
    You are welcome @strasvery :) I will say you get better at being thorough the more you examine the questions that trip you up.
Sign In or Register to comment.